Torque on Rod Due to Normal Force at a Hinge

In summary, the problem involves a diagram with a rod attached to a hinge and a support stick. The task is to determine if there is any torque due to the normal force from the hinge once the support stick is removed, and what the normal force would be at the hinge. The equations used are ##F_{net} = ma##, ##\tau_{net} = I\alpha##, and ##\tau = Frsin(\phi)##, where ##\phi## is the angle between the force and the displacement vector. The solution measures torque from the hinge axis and states that the torque is ##\tau_{net} = Mgcos(\theta)\frac{L}{2}##. However, it is unclear if the
  • #1
erfz

Homework Statement


In this diagram,
free_f2gif.gif

I wondered if there is any torque due to the normal force from the hinge, once the support stick is removed. I also want to know what the normal force would be at the hinge. The cup and ball are to be ignored here (essentially massless).

Homework Equations


##F_{net} = ma##
##\tau_{net} = I\alpha##
##\tau = Frsin(\phi)##, where ##\phi## is the angle between F and r

The Attempt at a Solution


My assumption would be yes: there is a torque due to the normal force acting straight up from the hinge on the point touching the table. However, the solution to the problem measures torque from the hinge axis (going in and out of the page), and states that the torque is ##\tau_{net} = Mgcos(\theta)\frac{L}{2}##.
I'm confused by this, but I feel that there are two possibilities here:
  1. The torque due to the hinge is somehow 0
  2. The "net" in the expression ##\tau_{net} = I\alpha## depends on the axis it is measured with respect to, just like ##\tau = Frsin(\phi)##. That is to say that the only torques that contribute to the net are those not acting at the axis of choice.
Is one of my suppositions correct?

As for the normal force at the hinge, I was thinking that I could say that ##F_{net, ~vertical} = Mg - N## at the center of mass? This happening at only the center of mass is just a guess. So then, if I go through ##\tau = I\alpha = \frac{1}{3}ML^2 \alpha = Mgcos(\theta)\frac{L}{2}##, I can derive that ##\alpha = \frac{3g}{2L}cos(\theta)## and that the linear acceleration down from this is ##a_{down} = \alpha cos(\theta) \frac{L}{2}##. Using this for the ##F_{net}##, I get that ##a_{down} = g - N##, so ##N = g - a_{down}##.
Is this reasoning correct here?

Thanks!
 

Attachments

  • free_f2gif.gif
    free_f2gif.gif
    6.5 KB · Views: 2,255
Last edited by a moderator:
Physics news on Phys.org
  • #2
erfz said:
torque due to the normal force
Torque due to a force is only meaningful in respect of a specified axis. Which axis do you mean?
In particular, if a force acts at a point P then it creates no torque about P.
 
  • #3
haruspex said:
Torque due to a force is only meaningful in respect of a specified axis. Which axis do you mean?
In particular, if a force acts at a point P then it creates no torque about P.
I see. I meant the axis at the hinge, going in and out of the page.
So this means that supposition 2 is the accurate one.
 
  • #4
erfz said:
I see. I meant the axis at the hinge, going in and out of the page.
So this means that supposition 2 is the accurate one.
Yes, but they are both true.
 
  • #5
haruspex said:
Torque due to a force is only meaningful in respect of a specified axis. Which axis do you mean?
In particular, if a force acts at a point P then it creates no torque about P.
haruspex said:
Yes, but they are both true.
Fantastic!
Is there anything wrong you see with my reasoning of the normal force?
 
  • #6
erfz said:
Is there anything wrong you see with my reasoning of the normal force?
There seem to be some other forces with vertical components, from the stick, the ball and the cup.
But you have not actually stated the problem that goes with the diagram, so it is hard to say. You mention accelerations, for example.
 
  • #7
haruspex said:
There seem to be some other forces with vertical components, from the stick, the ball and the cup.
But you have not actually stated the problem that goes with the diagram, so it is hard to say. You mention accelerations, for example.
Sorry I should've made it more clear that I'm only considering the system after the supporting stick is removed and that the ball and cup are massless.
I'm really only considering the rod and the hinge for what I'm asking.
 
  • #8
erfz said:
Sorry I should've made it more clear that I'm only considering the system after the supporting stick is removed and that the ball and cup are massless.
I'm really only considering the rod and the hinge for what I'm asking.
Ah, yes, I did see that earlier but when I came back to the thread I forgot. A risk with multitasking.
erfz said:
at the center of mass?
For linear force analysis, the line of action does not matter, only the direction.

The rest of your work looks fine, except that you forgot to divide N by M at the end.
 
  • #9
haruspex said:
For linear force analysis, the line of action does not matter, only the direction.
Sorry, I'm not sure what you mean by this. Do you mean the analysis does not require forces be considered at the center of mass? How can that be if different points on the rod have different linear accelerations downward?

haruspex said:
The rest of your work looks fine, except that you forgot to divide N by M at the end.
Haha silly error on my part.
 
  • #10
erfz said:
Sorry, I'm not sure what you mean by this. Do you mean the analysis does not require forces be considered at the center of mass? How can that be if different points on the rod have different linear accelerations downward?
If a force F is exerted on a rigid mass m (floating in space, say) then the acceleration of its mass centre is F/m. The force can be exerted at any point and at any angle. Its line of action does not need to pass through the mass centre.
 
  • #11
haruspex said:
If a force F is exerted on a rigid mass m (floating in space, say) then the acceleration of its mass centre is F/m. The force can be exerted at any point and at any angle. Its line of action does not need to pass through the mass centre.
That's great to know, but I'm not sure why you bring it up. Is there something in my work that contradicts this statement? My main assumption was that the force analysis is to be considered at the center of mass, which is important since every point on the bar has different downward acceleration (proportional to the distance from the hinge). That is to say that ##M_{cm} a_{cm, ~down} = M_{cm}g - N##.
 
  • #12
erfz said:
That's great to know, but I'm not sure why you bring it up. Is there something in my work that contradicts this statement? My main assumption was that the force analysis is to be considered at the center of mass, which is important since every point on the bar has different downward acceleration (proportional to the distance from the hinge). That is to say that ##M_{cm} a_{cm, ~down} = M_{cm}g - N##.
Because you wrote:
erfz said:
I was thinking that I could say that ##F_{net, ~vertical} = Mg - N## at the center of mass? This happening at only the center of mass is just a guess.
 
  • #13
So is that to say that I was correct in my assumption?
 
  • #14
erfz said:
So is that to say that I was correct in my assumption?
Yes.
 
  • #15
haruspex said:
Yes.
Wonderful. Thank you for all your help!
 

1. What is torque on a rod?

Torque is a measure of the force that causes an object to rotate around an axis. In the context of a rod, it refers to the force that causes the rod to rotate around a fixed point, such as a hinge.

2. What is the normal force at a hinge?

The normal force at a hinge is the force that acts perpendicular to the surface of the hinge. In the case of a rod, this force is exerted by the hinge on the rod and is necessary to keep the rod in place.

3. How does the normal force at a hinge affect torque on a rod?

The normal force at a hinge can affect the torque on a rod by creating a moment arm, which is the distance between the hinge and the point where the force is applied. The larger the moment arm, the greater the torque on the rod.

4. How is torque on a rod calculated in relation to the normal force at a hinge?

The torque on a rod due to the normal force at a hinge can be calculated by multiplying the magnitude of the normal force by the length of the moment arm. This can be represented mathematically as T = F x r, where T is torque, F is the normal force, and r is the moment arm.

5. Why is understanding torque on a rod due to normal force at a hinge important?

Understanding torque on a rod due to normal force at a hinge is important because it allows us to predict how forces at a hinge will affect the rotation of the rod. This information is useful in designing and analyzing mechanical systems that involve hinges and rotating rods, such as doors, gates, and other structures.

Similar threads

  • Introductory Physics Homework Help
Replies
7
Views
313
  • Introductory Physics Homework Help
2
Replies
42
Views
2K
Replies
7
Views
281
  • Introductory Physics Homework Help
Replies
4
Views
337
  • Introductory Physics Homework Help
Replies
11
Views
227
  • Introductory Physics Homework Help
Replies
7
Views
2K
  • Introductory Physics Homework Help
3
Replies
95
Views
4K
  • Introductory Physics Homework Help
Replies
4
Views
972
  • Introductory Physics Homework Help
Replies
9
Views
2K
  • Introductory Physics Homework Help
Replies
1
Views
3K
Back
Top